LSAT and Law School Admissions Forum

Get expert LSAT preparation and law school admissions advice from PowerScore Test Preparation.

 Administrator
PowerScore Staff
  • PowerScore Staff
  • Posts: 8916
  • Joined: Feb 02, 2011
|
#22907
Complete Question Explanation

Justify the Conclusion. The correct answer choice is (C)

The author of this stimulus makes the following argument:

Premise: One can do more calculations with a computer than with an adding machine.
  • use computer .......... more calculations per hour
Conclusion: Therefore an accountant who uses a computer can earn more.
  • use computer .......... earn more
So, we know that an accountant who uses a computer can do more calculations. According to the author, this means that the accountant can earn more. That means that we can put together a nearly perfectly prephrased answer to this question: the correct answer choice will almost certainly link the two rogue elements above. The overlapping element is "use computer," so the correct answer choice should link "more calculations per hour" with "earn more."

Answer choice (A): There is no comparison in the stimulus regarding the relative popularity among accountants of adding machines versus computers, so this answer choice is incorrect.

Answer choice (B): The stimulus regards efficiency more than number of work hours. This answer choice does not provide the necessary link, so it cannot be the correct answer to this Justify question.

Answer choice (C): This is the correct answer choice, and it does in fact link the rogue elements necessary: the more calculations an accountant performs, the more that accountant will earn:
  • use computer .......... more calculations per hour .......... earn more
This choice justifies the conclusion that an accountant who uses a computer will earn more than an accountant who uses an adding machine.

Answer choice (D): The author's conclusion is that an accountant can earn more, and this answer choice would actually weaken that argument, so it cannot be the choice which justifies the author's conclusion.

Answer choice (E): This general assertion does is not necessary to justify the author's argument, which requires that accountants who do more calculations earn more money.
 lathlee
  • Posts: 652
  • Joined: Apr 01, 2016
|
#43275
Even though I have no problem with the answer to C being correct. this is one thing that bothers me: Question Type

Question: Which one of the following is an assumption that would make the conclusion in the passage a logical one?

I feel this Q type character is a hybrid half/half, Half Assumption and Half Justify characteristics.

Also, the answer of C is as much as supporter/ Link Assumption as much as Justifying conclusion mechanism.

Wham am i not seeming?
 Adam Tyson
PowerScore Staff
  • PowerScore Staff
  • Posts: 5153
  • Joined: Apr 14, 2011
|
#43681
This one is a Justify all the way, lathlee, and the clue is that the stem asks about making the conclusion a logical one. It isn't asking which of the following answers MUST be assumed. Rather, it is asking which of the following, IF assumed, would justify the conclusion. They managed to ask it without using the word "if", but it is implied nonetheless. The hint there is the use of the word "would", which implies "if true". You can imagine this question being modified just slightly to read: "Which one of the following is an assumption that would, if true, make the conclusion in the passage a logical one?" The meaning of the stem hasn't changed one bit! The "if true" was implied all along, and true Assumption questions NEVER have an "if true" aspect to them. They are things that MUST be assumed by the author.

Good job getting the right answer, however you got there!
 yupyup
  • Posts: 10
  • Joined: Jul 27, 2019
|
#71859
If this question is Justify, then I'm confused as to what makes question 22 from PT 12 Sec. 1 an assumption question? That question's stem says "Which one of the following is an assumption that would allow the company president's conclusion to be properly drawn?" And the explanation for that question in this forum stated that it was an assumption question. How is that stem different from this stem?
Adam Tyson wrote:This one is a Justify all the way, lathlee, and the clue is that the stem asks about making the conclusion a logical one. It isn't asking which of the following answers MUST be assumed. Rather, it is asking which of the following, IF assumed, would justify the conclusion. They managed to ask it without using the word "if", but it is implied nonetheless. The hint there is the use of the word "would", which implies "if true". You can imagine this question being modified just slightly to read: "Which one of the following is an assumption that would, if true, make the conclusion in the passage a logical one?" The meaning of the stem hasn't changed one bit! The "if true" was implied all along, and true Assumption questions NEVER have an "if true" aspect to them. They are things that MUST be assumed by the author.

Good job getting the right answer, however you got there!
User avatar
 Dave Killoran
PowerScore Staff
  • PowerScore Staff
  • Posts: 5853
  • Joined: Mar 25, 2011
|
#72356
Hi yupyup,

This is a good example of how very small changes in language can affect outcomes. Let's look at the key difference, with italics added:

  • This stem: "Which one of the following is an assumption that would make the conclusion in the passage a logical one?

    The stem from the question you cite: "Which one of the following is an assumption that would allow the company president's conclusion to be properly drawn?"
That difference between make, which works on the sufficiency side, and allow, which doesn't imply sufficiency, is enough to send one to the Justify classification and the other to the Assumption classification.

Thanks!

Get the most out of your LSAT Prep Plus subscription.

Analyze and track your performance with our Testing and Analytics Package.